1
$\begingroup$

I want to prove the following:

Let $G$ be a finite abelian $p$-group that is not cyclic. Let $L \ne {1}$ be a subgroup of $G$ and $U$ be a maximal subgroup of L then there exists a maximal subgroup $M$ of $G$ such that $U \leq M$ and $L \nleq M$.

Proof. If $L=G$ then we are done.Suppose $L \ne G$ . Let $|G|=p^{n}$ then $|L|=p^{n-i}$ and $|U|=p^{n-i-1}$ for some $0< i < n$. There is $x_{1} \in G$ such that $x_{1} \notin L$. Thus $|U\langle x_{1}\rangle|=p^{n-i}$ and does not contain L. There is $x_{2} \in G$ such that $x_{2} \notin L$ and $x_{2} \notin |U\langle x_{1}\rangle|$. Thus $|U\langle x_{1}\rangle\langle x_{2}\rangle|=p^{n-i+1}$. Continuing like this, we get $|U\langle x_{1}\rangle \langle x_{2}\rangle\cdots \langle x_{i}\rangle|=p^{n-1}$ is a maximal subgroup of $G$. The problem is, I am not sure that this subgroup does not contain $L $.

Thanks in advance.

$\endgroup$
2
  • 1
    $\begingroup$ Just to add to Geoff Robinson's answer below. A counterexample can be obtained by taking $p=2$, $G$ cyclic of order $4$, $U = \{1\}$ and $L$ the unique proper non-trivial subgroup of $G$. $\endgroup$
    – user91132
    May 25, 2012 at 16:54
  • $\begingroup$ $G$ can not be cyclic by assumption. We can easily find counterexample by Geoff Robinson's answer. Let $G=\langle a \rangle \times \langle b \rangle$, where $|a|=4$. Then $a^2$ is in Frattini subgroup of $G$. Hence $L=\langle a^2 \rangle$ has no complement. $\endgroup$
    – Wei Zhou
    May 26, 2012 at 0:46

2 Answers 2

4
$\begingroup$

This is false in general. Consider the case (which you can reduce to by isomorphism theorems) that $U =1$ and $L$ then necessarily has order $p.$ You are asking for the existence of a complement to $L$ in $G$, for you would have $G = L \times M$ if there were such a maximal subgroup $M.$ There is such a subgroup $M$ if and only if $L$ is not contained in $\Phi(G),$ the Frattini subgroup of $G.$ More precisely, in general you are guaranteed to find the maximal subgroup you want if and only if $L/U$ is not contained in the Frattini subgroup of $G/U.$ Given a subgroup $U$, unless $G/U$ is elementary Abelian (ie an Abelian $p$-group of exponent $p )$, you can always find a subgroup $L$ containing $U$ with $[L:U] = p$ such that every maximal subgroup of $G$ containing $U$ will also contain $L.$

$\endgroup$
0
$\begingroup$

A s noted Prof. Robinson, this is false. However, this is true iff $L\not\le M\Phi(G)$. Indeed, let $\bar G=G/M\Phi(G)$; then $\bar L$ is a direct factor of $\bar G$ of order $p$. If $\bar G=\bar L\times\bar U$. Then $U$ is a maximal subgroup of $G$ and $U\cap L=M$.

$\endgroup$

Your Answer

By clicking “Post Your Answer”, you agree to our terms of service and acknowledge you have read our privacy policy.

Not the answer you're looking for? Browse other questions tagged or ask your own question.